5
$\begingroup$

Suppose that $R$ is a (commutative, unital) ring and that $A$ is a (commutative, unital) $R$-algebra that is projective of constant rank $n$ as an $R$-module. Then $A$ has a "determinant line bundle" $\bigwedge^n_R A$, which is projective of constant rank $1$ as an $R$-module.

Now if $A$ has an $A$-module $M$ that is projective of rank $m$, then we may use it to produce a rank-1 $R$-module in two different ways:

  1. We can take the determinant line bundle of $M$ as an $A$-module to get $\bigwedge^m_A M$. We may then regard this rank-$1$ $A$-module as a rank-$n$ $R$-module and take its determinant line bundle to get $\bigwedge_R^n\bigl(\bigwedge_A^m M\bigr)$.
  2. We can regard $M$ immediately as a rank-$mn$ $R$-module and take its determinant line bundle: $\bigwedge_R^{mn} M$.

These are not in general isomorphic. If $A$ has an $R$-basis $\{x_1,\dots,x_n\}$ and $M$ has an $A$-basis $\{y_1,\dots,y_m\}$, then we would want to match up the basis element $x_1y_1 \wedge x_1y_2 \wedge \dots\wedge x_ny_m$ for line bundle #1 with the basis element $x_1(y_1\wedge\dots\wedge y_m) \wedge x_2(y_1\wedge\dots\wedge y_m)\wedge \dots\wedge x_n(y_1\wedge\dots\wedge y_m)$ for line bundle #2. However, this matching is not invariant under change-of-basis: There are $m$ factors of each $x_i$ in the first element but only one factor of each in the second element, so if we scale $x_1$ by a unit the two elements would no longer correspond.

It seems to me that the correct formula should be $$\bigwedge^{mn}_R M \cong \bigwedge_R^n\left(\bigwedge_A^m M\right) \otimes \left(\bigwedge_R^n A\right)^{\otimes(m-1)},$$ so that the number of factors of each $x_i$ and $y_j$ on each side would match, but that is not a proof. Does anyone have a simple proof or reference for this result? It seems an identity relating the pushforward of a determinant line bundle with the determinant line bundle of a pushforward would be pretty standard.

$\endgroup$
6
  • $\begingroup$ I suspect you get such a formula by unpackaging Grothendieck-Riemann-Roch (but I didn't try it). $\endgroup$
    – Anonymous
    Dec 19, 2020 at 16:37
  • $\begingroup$ @Anonymous I'm having trouble seeing the connection — could you elaborate? $\endgroup$ Dec 19, 2020 at 16:49
  • $\begingroup$ Interesting question! Do you expect the isomorphism to be canonical (in any sense)? $\endgroup$ Dec 19, 2020 at 19:22
  • 1
    $\begingroup$ Hi, @darijgrinberg ! Yes, I do expect it to be canonical, in the sense that when you pick bases for M and A, you can write down the isomorphism explicitly (because then they're both free) but the isomorphism you write down doesn't actually depend on the choice of bases. $\endgroup$ Dec 19, 2020 at 20:32
  • 1
    $\begingroup$ Maybe something like BCnrd's comments here mathoverflow.net/questions/44918/… (transitivity of norm)? $\endgroup$ Dec 19, 2020 at 20:49

1 Answer 1

3
$\begingroup$

Yes, the identity holds! Thanks to @user2831784 for providing a link to the reference "Nombres de Tamagawa et groupes unipotents en caractéristique p" by Joseph Oesterlé in Invent. math. 78, 13-88 (1984). There, section 4.2 of Chapter II has the proposition that the "norm of line bundles" operation $N_{A/R}$ satisfies

$$\bigwedge_R^{mn} M \cong \left(\bigwedge_R^n A\right)^{\otimes m}\otimes N_{A/R}\left(\bigwedge_A^m M\right).$$ This is almost what we want: if we apply the above identity not to $M$ but to $\bigwedge_A^m M$, we get $$\bigwedge_R^n\left(\bigwedge_A^m M\right)\cong \left(\bigwedge_R^n A\right)^{\otimes 1}\otimes N_{A/R}\left(\bigwedge_A^m M\right).$$

Comparing these two identities gives the desired isomorphism

$$\bigwedge_R^{mn} M \cong \bigwedge_R^n\left(\bigwedge_A^m M\right) \otimes \left(\bigwedge_R^n A\right)^{\otimes (m-1)}.$$

Thanks for your help!

$\endgroup$

Your Answer

By clicking “Post Your Answer”, you agree to our terms of service and acknowledge you have read our privacy policy.

Not the answer you're looking for? Browse other questions tagged or ask your own question.